Aria invested $40,000 in an account paying an interest rate of 8\tfrac{3}{8}8 8 3 ​ % compounded daily. Olivia invested $40,000 in an account paying an interest rate of 8\tfrac{1}{8}8 8 1 ​ % compounded quarterly. To the nearest hundredth of a year, how much longer would it take for Olivia's money to triple than for Aria's money to triple?

Answers

Answer 1

To the nearest hundredth of a year (period), it will take Olivia's money 0.54 years more to triple than Aria's money to triple.

How the period is determined?

The additional period it will take Olivia's money to triple than Aria's is determined as the difference between the two investment periods.

The investment periods are computed using an online finance calculator as follows:

Aria Investment:

I/Y (Interest per year) = 8.375%

PV (Present Value) = $40,000

PMT (Periodic Payment) = $0

FV (Future Value) = $-120000

Results:

Number of Periods (N) = 4,788.531

= 13.119 years (4,788.531/365) or 13 years and 1 month

Total Interest = $80,000.00

Olivia's Investment:

I/Y (Interest per year) = 8.125%

PV (Present Value) = $40,000

PMT (Periodic Payment) = $0

FV (Future Value) = $-120000

Results:

N = 54.633

= 13.65825 years (54.633 quarters/4) or 13 years and 8 months

Total Interest $80,000.00

The difference in investment period:

13.66 - 13.12

= 0.54 years

Learn more about determining the investment period at https://brainly.com/question/20392017

#SPJ1


Related Questions

Help please!!!

The finance charge on your credit card was supposed to be $12.22. Instead you were charged $19.54.
Find the absolute error and the relative error in the finance charge

Answers

Taxes and federal made you get charged like that but you can divide it

Answer:The absolute error is that they said that the finance charge will be $12.22 but the charge ends up being $19.54 . As for the relative error is that they said that the charge will be $12.22 but the charge was $19.54 which means they were unsure what the charge was and failed to check what it actually was.

Step-by-step explanation:

Treys grandmother has a framed picture with side lengths that are 12 inches. Trey wants to know how much wall space is needed for the picture.

Answers

Answer:

144 inches

Step-by-step explanation:

Answer:

144 inches :)

Step-by-step explanation:

The school sold 300 tickets to the theatre performance. They sold adult tickets for $9 each and student tickets for $6 each. They made a total of $2, 190. How many students were in attendance?
I need help please, I need to show work and it is due tonight.

Answers

Answer:

Your answer is.....

170 students

Mark it as brainlist answer . follow me for more answer.

Step-by-step explanation:

Three angles in a triangle measure x°, (4x+12)° and (3x+16)°. What is the measure of the largest angle in this triangle?

Answers

Answer:

Step-by-step explanation:

90°

Chad measured a house and its lot and made a scale drawing.
The backyard is 17 inches long
in the drawing.
The actual yard is 68 feet long.
What scale did Chad use for the drawing?

1 inch __ feet

Answers

Answer: 4 maybe because if its 17 inches divide 68 by 17 and it should give you your answer

3,864,950 rounded to the nearest ten thousand

Answers

Answer:

3,870,000

Step-by-step explanation:

The 5 rounds the 9 into a 10

The 10 rounds the 4 into a 5

The 5 rounds the 6 into a 7

Put them all together and you get 3,870,000

I hope this helps, and have a great day! :)

Lines m and n are parallel Line m is described by the equation y = 40 +2. Line n contains the point (1, -3). Which of the following is an equation for linen?
A Oy-3= (2+1)
B. Oy+ 3 = 4(x - 1)
M
cOy-3 = 4(2+1)
DOy+3 = (- 1)

Answers

Answer:

B)Oy+ 3 = 4(x - 1)

Equation of the straight line   y +3 = 4( x-1)

Step-by-step explanation:

Explanation:-

Step(i):-

Given that the equation   y = 4x +2

and given that the point (1,-3)

The equation of the parallel line to the given line

                       ax +by +k=0

                       4x-y +k=0

This line is passing through the point (1,-3)

                  ⇒ 4(1) -(-3) +k=0

                  ⇒   4+3 +k=0

                          7 +k=0

                             k =-7

The equation of the Parallel line  4x-y -7 = 0

Step(ii):-

Given that the slope m = 4 and the point (1,-3)

Equation of the straight line

             y - y₁ = m(x-x₁)

             y -(-3) = 4( x-1)

            y +3 = 4( x-1)

The points (2, 8) and (1, 2) fall on a particular line. What is its equation in slope-intercept form?

Answers

Answer:

y = 6x - 4

Step-by-step explanation:

(2, 8) & (1, 2)

First you want to find the slope of the line that passes through these points. To find the slope of the line, we use the slope formula: (y₂ - y₁) / (x₂ - x₁)

Plug in these values:

(2 - 8) / (1 - 2)

Simplify the parentheses.

= (-6) / (-1)

Simplify the fraction.

-6/-1

= 6

This is your slope. Plug this value into the standard slope-intercept equation of y = mx + b.

y = 6x + b

To find b, we want to plug in a value that we know is on this line: in this case, I will use the second point (1, 2). Plug in the x and y values into the x and y of the standard equation.

2 = 6(1) + b

To find b, multiply the slope and the input of x(1)

2 = 6 + b

Now, subtract 6 from both sides to isolate b.

-4 = b

Plug this into your standard equation.

y = 6x - 4

This is your equation.

Check this by plugging in the other point you have not checked yet (2, 8).

y = 6x - 4

8 = 6(2) - 4

8 = 12 - 4

8 = 8

Your equation is correct.

Hope this helps!

3.- Una persona pinta el interior de un cuadrado de lado 15m, si antes de pintarlo
comete un error al medir la longitud de los lados de 0.2m, resolver por diferencial
(a).- ¿Cuál es el error máximo en el área de pintado (cuanto de diferencia de área
pinto más debido al error).​

Answers

Answer: SOrry no espanypol

Step-by-step explanation:

Find the percent of change

1970's: $0.79
2010's: $3.72

Answers

Answer:

471%

Step-by-step explanation:

If you divide 3.72 by .79 you get 4.7088...

Multiply by 100 to get your percentage is 470.88...

Rounded to nearest percentage is 471

hope this helps

Sadie swam x laps in the pool. Eric swam 3/5 more than that. Write an expression for the number of laps Eric swam

Answers

Answer:

y = 3/5 + x

'y' = # laps Eric swam

Step-by-step explanation:

Answer:

                                                             .

Step-by-step explanation:

                                                                  .

If you answer these questions i will give you free point's. (Answer all of them pls)


1. I am 50% of a quarter of 32.
2. I am two thirds of 60.
3. I am exactly half-way between 25% of 80 cm and 40% of 120cm.
4. I am one-quarter of 50% of 88kg.
5. I am 3% of half of 600 pounds.
6. I am exactly halfway between one fifth of 25cm and 2% of 300 cm.
7. I am greater that 0.6 kg, less than one-third of 6 kg, and equal to four times 10% of 3kg.

Answers

Answer:

1. 4

2.40

3. 28

4.11

5.12

6. 1

7. 1200

Step-by-step explanation:

I used • instead of x to show you multiply.

1. 1/4 (one quarter) of 32 = 8.
50% of 8 = 4

2. 2/3• 60 = 120/3 = 40

3. 25% = 0.25, 40% = 0.40
So 0.25 • 80= 20
0.40 • 120 = 48
Then subtract: 48-20 = 28. Half of 28 is 14. Add that 14 to 20, and your answer is 34. It will probably be helpful to either write that on a number line, or picture it on a number line.

4. 50% of 88kg is half of that, which is 44. One quarter = 1/4, so 44/4 = 11

5. Half of 600 pounds is 300 pounds. 3% = 3/100 = 0.03. Multiply that by 300 to get your answer. 300 • 0.03 = 9

6. 1/5 of 25cm = 25/5= 5
2% of 300 = 300• 0.02= 6
What is exactly halfway between 5 and 6? 5.5

7. Start this one with the last statement. 10% (0.10) of 3kg = 3•0.10= 0.3. Four times that = 4•0.3= 1.2

Then check: is that greater than 0.6? Yes.

One third of 6kg = 6/3= 2

Is 1.2 less than 2? Yes.

HELP WILLING TO CASHAPP if answered quick and correctly + 100 points The points plotted below on the graph of a polynomial which of the following X values is the best approximation of a root of the polynomial? Check all that apply
X=-0.21
X=-4.6
X=-2.3
X=3
X=1
X=-3.5

Answers

first 1

4th one

i think sorry if its wrong

Answer:

Step-by-step explanation:

Did you ever find out the answer????

Help me pls I don't know what I'm doing ​

Answers

Answer:

B. 3.6

Step-by-step explanation:

I have no idea if this is right but my logic is 3.6 - 3.6 = 0

Answer:

B. 3.6

Step-by-step explanation:

The equation is

w - 3.6 = 0

That means start with some number. Then subtract 3.6 from that number. The result is 0. What number is it that when you subtract 3.6 from it you get 0? That number is 3.6 since 3.6 - 3.6 = 0. That means w, the unknown, must equal 3.6.

That is the explanation.

The way to solve the equation is to see what is being done to the unknown. The unknown here is w. On the left side, you see 3.6 being subtracted from w. In order to undo a subtraction, you must do the opposite operation which is an addition. In order to undo a subtraction of 3.6, you must add 3.6. The rule with equations is that you must do the same operation to both sides of the equation, so add 3.6 to both sides of the equation.

w - 3.6 = 0

Add 3.6 to both sides.

w - 3.6 + 3.6 = 0 + 3.6

w = 3.6

Answer: B. 3.6

what are integra in algebra ​

Answers

Answer:

this is what i found for ya:3

Step-by-step explanation:

In mathematics, specifically abstract algebra, an integral domain is a nonzero commutative ring in which the product of any two nonzero elements is nonzero. ... Noncommutative integral domains are sometimes admitted.

Answer:

That's the meaning of of integers in math and algebra: Whole numbers. No fractions allowed! Integers are counting numbers, and they include 0. ... In other words, integers can be negative, but they can't be fractional. Any number with a fraction – and that includes decimal fractions – is not an integer.

Step-by-step explanation:

Mia and her friends are planning a trip to the movies. They can go to the movies on Friday, Saturday, or Sunday, and there are 10 different movies at the theater. Which expression represents the number of different combinations of day and movie Mia and her friends can choose?

Answers

Answer:

30

Step-by-step explanation:

There are 3 days, and 10 movies.

3 x 10 = 30

20 PTS !!!!!!
HELP PLEASE

Answers

It is 53...... BODMAS


What is the value of f(-5)?

Answers

u have to plug in -5 as x in the function/equation

Answer:

None

Step-by-step explanation:

Cant solve it without an X value

What is the distance between points (-2, -5) and (-14, 4) on the coordinate plane

•9
•12
•14
•15

Answers

Answer:

15 units

Step-by-step explanation:

[tex]distance formula =\sqrt{(x_{2} -x_{1} )^{2} + (y_2 -y1)^{2} } } \\[/tex]

[tex]x_{1} = -2\\x_{2} = -14\\y_{1} = -5\\y_{2} = 4[/tex]

[tex]distance\\= \sqrt{(-14 - -2)^{2} + (4 - -5)^2 } \\= \sqrt{(-12)^2 + (9)^2} \\= \sqrt{144 + 81} \\= \sqrt{225} \\= 15\\Thus distance between the two points (-2,-5) and (-14,4) = 15 units[/tex]

Hope it helps....

Question 4(8 points)
Ralph receives money from his family members for his high school graduation. He
decides he wants to spend no more than $120.00 on movies to bring with him to
college. Each DVD is $10.00, and each Blu-Ray is $15.00. He wants to purchase at least
8 movies. Determine the following,
You are given this inequality to represent "he wants to purchase at least 8
movies": x+y 28
BLANK 1: Would this be the correct way to write this (yes or no)?
BLANK 2-4: Fill in the blanks for the second inequality (all three answers are whole
numbers):
ys
x+
BLANK 5 AND 6:
What label would go on the x-axis, if you graphed this?
What label would go on the y-axis, if you graphed this?
BLANK 7 AND 8:
If Ralph were to JUST purchase DVDs, what is the maximum amount he could buy?
If Ralph were to JUST purchase Blu-Rays, what is the maximum amount he could buy?

Answers

what is your name? Jan kskskksk ksksmskksnsns nsksmms a ksmsms

Now look at the 5-year performance of Amazon. What do you think explains the general upward trend of the stock price?

Answers

Answer:

expansion

Step-by-step explanation:

I believe that the biggest reason for the general upward trend in the stock price is expansion. Amazon has expanded their company immensely over the last 5 years. They have also expanded into different markets such as drones, gaming, cloud streaming, video streaming, etc. All of these expansions and acquisitions have been incredibly profitable for Amazon and therefore reflect on the stock price. This also gives investors faith that the future of the company will continue growing which causes them to buy more stocks and therefore increase the price.

Answer:

Step-by-step explanation:

James had 42 dollars. He bought his sister a gift that cost x dollars. Write an expression representing how much money James has now.

Answers

Answer:

42 dollars -x......................

The volume of a cube, in cubic centimeters, is given by the function V(x) = x3, where x is the side length of the cube in centimeters. Write a new function for the volume of the cube in cubic millimeters.

Answers

I think we can all agree that 1 centimeter=10millimeters.

And if x is the side in centimeters then 10x will be the side in millimeters.

We just need to rewrite x^3 into (10x)^3.

Because you need to do the exponent to all part of the ( )...

(10x)^3=1000x^3

V(x)=1000x^3 cubic millimeters is the answer

Hence, a new function for the volume of the cube in cubic millimeters is [tex]V(x)=1000x^3[/tex].

What is the volume of cube?

The volume of a cube is defined as the total number of cubic units occupied by the cube completely.

A cube is a three-dimensional solid figure, having six square faces. Volume is nothing but the total space occupied by an object.

Here given that,

The volume of a cube, in cubic centimeters, is given by the function [tex]V(x) = x^3[/tex], where [tex]x[/tex] is the side length of the cube in centimeters.

As, [tex]1[/tex] centimeter=[tex]10[/tex]millimeters.

If the side is [tex]x[/tex] then the side in millimeter is [tex]10x[/tex].

So, we have [tex]x^3[/tex] which would be [tex](10x)^3[/tex]

[tex](10x)^3=1000x^3[/tex]

Then,

[tex]V(x)=1000x^3[/tex] cubic millimeters.

Hence, a new function for the volume of the cube in cubic millimeters is [tex]V(x)=1000x^3[/tex].

To know more about the volume of a cube

https://brainly.com/question/16642309

#SPJ2

Plz help. One winter day, the temperature in Houston is 12 degrees Celsius. Find the temperatures in these other cities.
A) In Minneapolis, it is 30 degrees colder than it is in Houston.

B) In Fairbanks, it is 9 degrees colder than it is in Minneapolis

C) Write an addition equation that represents the relationship between the temperature in Houston and the temperature in Fairbanks. (Hint: Equation means it must be solved; so it has an equal symbol)

Answers

A= -18
B= -27
12-39=-27
These should be the answers

What percent is represented by the region that is shaded? Exact
answers only (no rounding).

Answers

I jus sayin the park with the woods on the woods at the mall

Answer:

64 percent

Step-by-step explanation:

because there is 24 sq and 100 divided by 24 is 4, 4 times 16 is 64

(0) A baker puts 10 kg of sugar equally into 8 bags. How much sugar is there in
each bag?​

Answers

Answer:

1.25 kg of sugar in each bag

Step-by-step explanation:

10/8 = 1.25

Please give a 5 star rating if this helped!

Tanner had $42.80 and spent 20% of his money on a book including tax. If he paid with a $10 bill, how much change did Tanner get back?

A. $7.28
B. $6.24
C. $1.44
D. $17.54

Answers

C.
42.8 x 0.2 = 8.56
10 - 8.56 = 1.44

1. You are baking cookies to bring to school for a bake sale. You have already finished 22 cookies, and you can bake 12 at a time on your cookie sheet. If you want to bake at least 100 cookies, how many more cookie sheets do you have to bake?

Answers

Answer:

pp

Step-by-step explanation:

pp

you need to bake 6 more sheets

help me plzzzzzzzzzzzzzzzzzzzzzzzzzzzzzzzzzzzzzzzzzzzzzzzzzzzzzzzzzzzzzzzzzzzzz

Answers

The first option, as it positively supports the claim stated.

Answer:

"Playing video games has been shown to enhance memory and even increase brain volume"

Step-by-step explanation:

The claim is approves the usage of video games.

The first piece of evidence supports the usage, citing how video games can "enhance memory" which is positive.

The second piece of evidence does the opposite and disapproves video games, citing them as an "addiction" which is negative.

(a) The area of a rectangular garden is 7371 m².
If the width of the garden is 81 m, what is its length?
What is the Length of the garden:

Answers

Answer:

length = 91 m

Step-by-step explanation:

area = length * width

so,

7371 = length * 81

cancel out 81 by dividing on both sides

91 = length

Other Questions
subtract (2y + 3x-x) from (5x-y2-2x) Describe the principle of Federalism and describe some of the shared powers between the federal and state governments. What did the Aztecs believe would happen if the gods were not fed? Please answer these computer fill in the blanks1) operators are the _______________ used in formulas to perform a particular calculation 2) the____________feature of Excelhelps to copy the formula quickly3) Excel provides over ________________used to perform calculations4) ____________ category of functions includes functions for calculation, and tracking principal and interest 5) ___________ functions returns one of two results you specify based on whether the value is TRUE or FALSE Honeybees eat nectar found in flowers. In the process of gathering nectar, bees carry pollen fromflower to flower. This allows the flowering plants to reproduce. Based on this information, what is thebest classification for the relationship between honeybees and flowering plants?A.mutualismB.commensalismC.parasitismD.competition what do u mean by custom mail Gamma rays have the shortest wavelength. True or False 9. son algunos de los elementos principales de la asistencia en la dispensacin:* the balance in the supplies account, before adjustment at the end of the year is 2750. the inventory of supplies at the end of the year was determined to be 600. the estimated depreciation on equipment at the end of the year is 1600. journalize the adjusting entries required at the end of the year to recognize A. supplies used during the year and B. depreciation expense for the year. Abby got a summer job. The table shows how much money she will earn based on the number of hours she works.Hours Worked 20 30 40 50Dollars Earned 150 225 300 375Based on the table, how much money will she earn if she works 65 hours?O A $450.00B. $475.50C. $487.50D. $525.00 what is the radius of a circle with the ratio 1:4 when the smaller one is 12cm Alec is thirsty and wants to know how much milk can fit in his favorite mug. His mug is a cylinder and has a radius(r) of 2 inches and a height (h) of 8 inches. The formula for the volume of a cylinder is given.V = trhHow much milk can Alec's mug hold? What is the area of the circle below?27 m424 m?848 m?O 572 3 m0 729.1 m Which of these best represents a call to action?O A. Bright colors on a billboardO B. "Buy 1 Get One Free!"O C. Loud background music to a commercialO D. "Come in today!" A certain sum of money gives $ 510 as compound interest 12/ % per annum for 2 years. Find the simple interest on the same sum of money at the same rate for the same period of time. Find x. Round to the nearest tenth if necessary. You are trying to push a 30 kg canoe across a beach to get it to a lake. Initially, the canoe isat rest, and you exert a force over a distance of 3 m until it has a speed of 1.2 m/s.a. How much work was done on the canoe? b. The coefficient of kinetic friction between the canoe and the beach is 0.2. How much work was done by friction on the canoe? c. How much work did you perform on the canoe? d. What force did you apply to the canoe? What is the greatest common factor of 12^5 and 8^3 A flower vase bought for Rs. 72 and sold for Rs. 63. Find the loss percent. 3. Why might incidents of chaos and violence lead people to make impulsive and evenillegal decisions? Have you heard of more recent instances of plundering and disregardfor property during periods of violence and unrest?